Show that if $a$ has order $3bmod p$ then $a+1$ has order $6bmod p$.












2












$begingroup$


Show that if $a$ has order $3bmod p$ then $a+1$ has order $6bmod p$. I know I am supposed to use primitive roots but I think this is where I am getting caught up. The definition of primitive root is "if $a$ is a least residue and the order of $abmod p$ is $phi(m)$ then $a$ is a primitive root of $m$. But I really am not sure how to use this to my advantage in solving anything.



Thanks!










share|cite|improve this question











$endgroup$












  • $begingroup$
    math.stackexchange.com/questions/332325/…
    $endgroup$
    – Zach Teitler
    Dec 14 '18 at 20:29
















2












$begingroup$


Show that if $a$ has order $3bmod p$ then $a+1$ has order $6bmod p$. I know I am supposed to use primitive roots but I think this is where I am getting caught up. The definition of primitive root is "if $a$ is a least residue and the order of $abmod p$ is $phi(m)$ then $a$ is a primitive root of $m$. But I really am not sure how to use this to my advantage in solving anything.



Thanks!










share|cite|improve this question











$endgroup$












  • $begingroup$
    math.stackexchange.com/questions/332325/…
    $endgroup$
    – Zach Teitler
    Dec 14 '18 at 20:29














2












2








2





$begingroup$


Show that if $a$ has order $3bmod p$ then $a+1$ has order $6bmod p$. I know I am supposed to use primitive roots but I think this is where I am getting caught up. The definition of primitive root is "if $a$ is a least residue and the order of $abmod p$ is $phi(m)$ then $a$ is a primitive root of $m$. But I really am not sure how to use this to my advantage in solving anything.



Thanks!










share|cite|improve this question











$endgroup$




Show that if $a$ has order $3bmod p$ then $a+1$ has order $6bmod p$. I know I am supposed to use primitive roots but I think this is where I am getting caught up. The definition of primitive root is "if $a$ is a least residue and the order of $abmod p$ is $phi(m)$ then $a$ is a primitive root of $m$. But I really am not sure how to use this to my advantage in solving anything.



Thanks!







number-theory






share|cite|improve this question















share|cite|improve this question













share|cite|improve this question




share|cite|improve this question








edited Aug 8 '13 at 19:05









MvG

30.8k449101




30.8k449101










asked Aug 8 '13 at 18:08









KathyKathy

1019




1019












  • $begingroup$
    math.stackexchange.com/questions/332325/…
    $endgroup$
    – Zach Teitler
    Dec 14 '18 at 20:29


















  • $begingroup$
    math.stackexchange.com/questions/332325/…
    $endgroup$
    – Zach Teitler
    Dec 14 '18 at 20:29
















$begingroup$
math.stackexchange.com/questions/332325/…
$endgroup$
– Zach Teitler
Dec 14 '18 at 20:29




$begingroup$
math.stackexchange.com/questions/332325/…
$endgroup$
– Zach Teitler
Dec 14 '18 at 20:29










2 Answers
2






active

oldest

votes


















9












$begingroup$

Note that if $a=1$, $a$ has order $1$. Thus, we can assume $ane1$. Furthermore, $pne2$ since no element mod $2$ has order $3$. Therefore, $-1ne1pmod{p}$.
$$
begin{align}
(a+1)^3
&=a^3+3a^2+3a+1\
&=1+3a^2+3a+1\
&=-1+3(1+a+a^2)\
&=-1+3frac{a^3-1}{a-1}\
&=-1
end{align}
$$

Therefore, $(a+1)^3=-1$ and $(a+1)^6=1$. (Shortened a la ccorn).
$$
begin{align}
(a+1)^2
&=a^2+2a+1\
&=a+(a^2+a+1)\
&=a+frac{a^3-1}{a-1}\
&=a\
&ne1
end{align}
$$

Therefore, $(a+1)^2ne1$.



Thus, $(a+1)$ has order $6$.






share|cite|improve this answer











$endgroup$









  • 1




    $begingroup$
    $(a+1)^6=1$ also follows easily from $(a+1)^3=-1$
    $endgroup$
    – ccorn
    Aug 8 '13 at 18:25






  • 1




    $begingroup$
    @ccorn: indeed it does, but I had already written the part for $(a+1)^6$. I guess I could shorten the answer. Thanks.
    $endgroup$
    – robjohn
    Aug 8 '13 at 18:27










  • $begingroup$
    With the edit made the (a+1)^{2} is not needed right? If it is I don't understand.
    $endgroup$
    – Kathy
    Aug 8 '13 at 19:19






  • 1




    $begingroup$
    @Kathy: we still have to show that $a+1$ doesn't have order $2$. We could argue that if $(a+1)^2=1$ and $(a+1)^3=-1$, then $a+1=-1$; that is, $a=-2$. Since $a^3=1$ we have that $-8equiv1pmod{p}$, which implies that $pmid9$, whence $p=3$. Since there are no elements of order $3$ mod $3$, we could stop. Personally, I think the argument in my answer is simpler.
    $endgroup$
    – robjohn
    Aug 8 '13 at 19:38










  • $begingroup$
    Is there a problem with this answer? Just wondering if the downvote means something needs fixing.
    $endgroup$
    – robjohn
    Jan 1 at 19:45



















1












$begingroup$

ord_$pa=3iff p$ divides $(a^3-1)=(a-1)(a^2+a+1)$



But $pnotmid (a-1)$ as ord_$pa=3 $



$displaystyleimplies a^2+a+1equiv0pmod p$
$displaystyleiff a(a+1)equiv-1pmod p$



Method $1:$



$implies a^3(a+1)^3equiv-1pmod pimplies (a+1)^3equiv-1$ and $(a+1)^6equiv1$



$implies $ord_$p(a+1)mid 6$ but does not divide $3$



If ord_$p(a+1)mid 2, (a+1)^2equiv1pmod p$ and $a^2(a+1)^2equiv1pmod pimplies a^2equiv1$ which is impossible as ord$_pa=3$



$implies $ord$_p(a+1)=6$



Method $2:$



$displaystyle a(a+1)equiv-1pmod piff a+1equiv (-a)^{-1}pmod p$



Again as $a^3equiv1pmod p, (-a)^3=-a^3equiv-1pmod pimplies (-a)^6equiv1implies $ord$_p(-a)=6$



Using this, ord$_p{(-a)^{-1}})=$ord$_p(-a)=6$






share|cite|improve this answer











$endgroup$













    Your Answer





    StackExchange.ifUsing("editor", function () {
    return StackExchange.using("mathjaxEditing", function () {
    StackExchange.MarkdownEditor.creationCallbacks.add(function (editor, postfix) {
    StackExchange.mathjaxEditing.prepareWmdForMathJax(editor, postfix, [["$", "$"], ["\\(","\\)"]]);
    });
    });
    }, "mathjax-editing");

    StackExchange.ready(function() {
    var channelOptions = {
    tags: "".split(" "),
    id: "69"
    };
    initTagRenderer("".split(" "), "".split(" "), channelOptions);

    StackExchange.using("externalEditor", function() {
    // Have to fire editor after snippets, if snippets enabled
    if (StackExchange.settings.snippets.snippetsEnabled) {
    StackExchange.using("snippets", function() {
    createEditor();
    });
    }
    else {
    createEditor();
    }
    });

    function createEditor() {
    StackExchange.prepareEditor({
    heartbeatType: 'answer',
    autoActivateHeartbeat: false,
    convertImagesToLinks: true,
    noModals: true,
    showLowRepImageUploadWarning: true,
    reputationToPostImages: 10,
    bindNavPrevention: true,
    postfix: "",
    imageUploader: {
    brandingHtml: "Powered by u003ca class="icon-imgur-white" href="https://imgur.com/"u003eu003c/au003e",
    contentPolicyHtml: "User contributions licensed under u003ca href="https://creativecommons.org/licenses/by-sa/3.0/"u003ecc by-sa 3.0 with attribution requiredu003c/au003e u003ca href="https://stackoverflow.com/legal/content-policy"u003e(content policy)u003c/au003e",
    allowUrls: true
    },
    noCode: true, onDemand: true,
    discardSelector: ".discard-answer"
    ,immediatelyShowMarkdownHelp:true
    });


    }
    });














    draft saved

    draft discarded


















    StackExchange.ready(
    function () {
    StackExchange.openid.initPostLogin('.new-post-login', 'https%3a%2f%2fmath.stackexchange.com%2fquestions%2f463010%2fshow-that-if-a-has-order-3-bmod-p-then-a1-has-order-6-bmod-p%23new-answer', 'question_page');
    }
    );

    Post as a guest















    Required, but never shown

























    2 Answers
    2






    active

    oldest

    votes








    2 Answers
    2






    active

    oldest

    votes









    active

    oldest

    votes






    active

    oldest

    votes









    9












    $begingroup$

    Note that if $a=1$, $a$ has order $1$. Thus, we can assume $ane1$. Furthermore, $pne2$ since no element mod $2$ has order $3$. Therefore, $-1ne1pmod{p}$.
    $$
    begin{align}
    (a+1)^3
    &=a^3+3a^2+3a+1\
    &=1+3a^2+3a+1\
    &=-1+3(1+a+a^2)\
    &=-1+3frac{a^3-1}{a-1}\
    &=-1
    end{align}
    $$

    Therefore, $(a+1)^3=-1$ and $(a+1)^6=1$. (Shortened a la ccorn).
    $$
    begin{align}
    (a+1)^2
    &=a^2+2a+1\
    &=a+(a^2+a+1)\
    &=a+frac{a^3-1}{a-1}\
    &=a\
    &ne1
    end{align}
    $$

    Therefore, $(a+1)^2ne1$.



    Thus, $(a+1)$ has order $6$.






    share|cite|improve this answer











    $endgroup$









    • 1




      $begingroup$
      $(a+1)^6=1$ also follows easily from $(a+1)^3=-1$
      $endgroup$
      – ccorn
      Aug 8 '13 at 18:25






    • 1




      $begingroup$
      @ccorn: indeed it does, but I had already written the part for $(a+1)^6$. I guess I could shorten the answer. Thanks.
      $endgroup$
      – robjohn
      Aug 8 '13 at 18:27










    • $begingroup$
      With the edit made the (a+1)^{2} is not needed right? If it is I don't understand.
      $endgroup$
      – Kathy
      Aug 8 '13 at 19:19






    • 1




      $begingroup$
      @Kathy: we still have to show that $a+1$ doesn't have order $2$. We could argue that if $(a+1)^2=1$ and $(a+1)^3=-1$, then $a+1=-1$; that is, $a=-2$. Since $a^3=1$ we have that $-8equiv1pmod{p}$, which implies that $pmid9$, whence $p=3$. Since there are no elements of order $3$ mod $3$, we could stop. Personally, I think the argument in my answer is simpler.
      $endgroup$
      – robjohn
      Aug 8 '13 at 19:38










    • $begingroup$
      Is there a problem with this answer? Just wondering if the downvote means something needs fixing.
      $endgroup$
      – robjohn
      Jan 1 at 19:45
















    9












    $begingroup$

    Note that if $a=1$, $a$ has order $1$. Thus, we can assume $ane1$. Furthermore, $pne2$ since no element mod $2$ has order $3$. Therefore, $-1ne1pmod{p}$.
    $$
    begin{align}
    (a+1)^3
    &=a^3+3a^2+3a+1\
    &=1+3a^2+3a+1\
    &=-1+3(1+a+a^2)\
    &=-1+3frac{a^3-1}{a-1}\
    &=-1
    end{align}
    $$

    Therefore, $(a+1)^3=-1$ and $(a+1)^6=1$. (Shortened a la ccorn).
    $$
    begin{align}
    (a+1)^2
    &=a^2+2a+1\
    &=a+(a^2+a+1)\
    &=a+frac{a^3-1}{a-1}\
    &=a\
    &ne1
    end{align}
    $$

    Therefore, $(a+1)^2ne1$.



    Thus, $(a+1)$ has order $6$.






    share|cite|improve this answer











    $endgroup$









    • 1




      $begingroup$
      $(a+1)^6=1$ also follows easily from $(a+1)^3=-1$
      $endgroup$
      – ccorn
      Aug 8 '13 at 18:25






    • 1




      $begingroup$
      @ccorn: indeed it does, but I had already written the part for $(a+1)^6$. I guess I could shorten the answer. Thanks.
      $endgroup$
      – robjohn
      Aug 8 '13 at 18:27










    • $begingroup$
      With the edit made the (a+1)^{2} is not needed right? If it is I don't understand.
      $endgroup$
      – Kathy
      Aug 8 '13 at 19:19






    • 1




      $begingroup$
      @Kathy: we still have to show that $a+1$ doesn't have order $2$. We could argue that if $(a+1)^2=1$ and $(a+1)^3=-1$, then $a+1=-1$; that is, $a=-2$. Since $a^3=1$ we have that $-8equiv1pmod{p}$, which implies that $pmid9$, whence $p=3$. Since there are no elements of order $3$ mod $3$, we could stop. Personally, I think the argument in my answer is simpler.
      $endgroup$
      – robjohn
      Aug 8 '13 at 19:38










    • $begingroup$
      Is there a problem with this answer? Just wondering if the downvote means something needs fixing.
      $endgroup$
      – robjohn
      Jan 1 at 19:45














    9












    9








    9





    $begingroup$

    Note that if $a=1$, $a$ has order $1$. Thus, we can assume $ane1$. Furthermore, $pne2$ since no element mod $2$ has order $3$. Therefore, $-1ne1pmod{p}$.
    $$
    begin{align}
    (a+1)^3
    &=a^3+3a^2+3a+1\
    &=1+3a^2+3a+1\
    &=-1+3(1+a+a^2)\
    &=-1+3frac{a^3-1}{a-1}\
    &=-1
    end{align}
    $$

    Therefore, $(a+1)^3=-1$ and $(a+1)^6=1$. (Shortened a la ccorn).
    $$
    begin{align}
    (a+1)^2
    &=a^2+2a+1\
    &=a+(a^2+a+1)\
    &=a+frac{a^3-1}{a-1}\
    &=a\
    &ne1
    end{align}
    $$

    Therefore, $(a+1)^2ne1$.



    Thus, $(a+1)$ has order $6$.






    share|cite|improve this answer











    $endgroup$



    Note that if $a=1$, $a$ has order $1$. Thus, we can assume $ane1$. Furthermore, $pne2$ since no element mod $2$ has order $3$. Therefore, $-1ne1pmod{p}$.
    $$
    begin{align}
    (a+1)^3
    &=a^3+3a^2+3a+1\
    &=1+3a^2+3a+1\
    &=-1+3(1+a+a^2)\
    &=-1+3frac{a^3-1}{a-1}\
    &=-1
    end{align}
    $$

    Therefore, $(a+1)^3=-1$ and $(a+1)^6=1$. (Shortened a la ccorn).
    $$
    begin{align}
    (a+1)^2
    &=a^2+2a+1\
    &=a+(a^2+a+1)\
    &=a+frac{a^3-1}{a-1}\
    &=a\
    &ne1
    end{align}
    $$

    Therefore, $(a+1)^2ne1$.



    Thus, $(a+1)$ has order $6$.







    share|cite|improve this answer














    share|cite|improve this answer



    share|cite|improve this answer








    edited Jan 1 at 20:27

























    answered Aug 8 '13 at 18:24









    robjohnrobjohn

    265k27303626




    265k27303626








    • 1




      $begingroup$
      $(a+1)^6=1$ also follows easily from $(a+1)^3=-1$
      $endgroup$
      – ccorn
      Aug 8 '13 at 18:25






    • 1




      $begingroup$
      @ccorn: indeed it does, but I had already written the part for $(a+1)^6$. I guess I could shorten the answer. Thanks.
      $endgroup$
      – robjohn
      Aug 8 '13 at 18:27










    • $begingroup$
      With the edit made the (a+1)^{2} is not needed right? If it is I don't understand.
      $endgroup$
      – Kathy
      Aug 8 '13 at 19:19






    • 1




      $begingroup$
      @Kathy: we still have to show that $a+1$ doesn't have order $2$. We could argue that if $(a+1)^2=1$ and $(a+1)^3=-1$, then $a+1=-1$; that is, $a=-2$. Since $a^3=1$ we have that $-8equiv1pmod{p}$, which implies that $pmid9$, whence $p=3$. Since there are no elements of order $3$ mod $3$, we could stop. Personally, I think the argument in my answer is simpler.
      $endgroup$
      – robjohn
      Aug 8 '13 at 19:38










    • $begingroup$
      Is there a problem with this answer? Just wondering if the downvote means something needs fixing.
      $endgroup$
      – robjohn
      Jan 1 at 19:45














    • 1




      $begingroup$
      $(a+1)^6=1$ also follows easily from $(a+1)^3=-1$
      $endgroup$
      – ccorn
      Aug 8 '13 at 18:25






    • 1




      $begingroup$
      @ccorn: indeed it does, but I had already written the part for $(a+1)^6$. I guess I could shorten the answer. Thanks.
      $endgroup$
      – robjohn
      Aug 8 '13 at 18:27










    • $begingroup$
      With the edit made the (a+1)^{2} is not needed right? If it is I don't understand.
      $endgroup$
      – Kathy
      Aug 8 '13 at 19:19






    • 1




      $begingroup$
      @Kathy: we still have to show that $a+1$ doesn't have order $2$. We could argue that if $(a+1)^2=1$ and $(a+1)^3=-1$, then $a+1=-1$; that is, $a=-2$. Since $a^3=1$ we have that $-8equiv1pmod{p}$, which implies that $pmid9$, whence $p=3$. Since there are no elements of order $3$ mod $3$, we could stop. Personally, I think the argument in my answer is simpler.
      $endgroup$
      – robjohn
      Aug 8 '13 at 19:38










    • $begingroup$
      Is there a problem with this answer? Just wondering if the downvote means something needs fixing.
      $endgroup$
      – robjohn
      Jan 1 at 19:45








    1




    1




    $begingroup$
    $(a+1)^6=1$ also follows easily from $(a+1)^3=-1$
    $endgroup$
    – ccorn
    Aug 8 '13 at 18:25




    $begingroup$
    $(a+1)^6=1$ also follows easily from $(a+1)^3=-1$
    $endgroup$
    – ccorn
    Aug 8 '13 at 18:25




    1




    1




    $begingroup$
    @ccorn: indeed it does, but I had already written the part for $(a+1)^6$. I guess I could shorten the answer. Thanks.
    $endgroup$
    – robjohn
    Aug 8 '13 at 18:27




    $begingroup$
    @ccorn: indeed it does, but I had already written the part for $(a+1)^6$. I guess I could shorten the answer. Thanks.
    $endgroup$
    – robjohn
    Aug 8 '13 at 18:27












    $begingroup$
    With the edit made the (a+1)^{2} is not needed right? If it is I don't understand.
    $endgroup$
    – Kathy
    Aug 8 '13 at 19:19




    $begingroup$
    With the edit made the (a+1)^{2} is not needed right? If it is I don't understand.
    $endgroup$
    – Kathy
    Aug 8 '13 at 19:19




    1




    1




    $begingroup$
    @Kathy: we still have to show that $a+1$ doesn't have order $2$. We could argue that if $(a+1)^2=1$ and $(a+1)^3=-1$, then $a+1=-1$; that is, $a=-2$. Since $a^3=1$ we have that $-8equiv1pmod{p}$, which implies that $pmid9$, whence $p=3$. Since there are no elements of order $3$ mod $3$, we could stop. Personally, I think the argument in my answer is simpler.
    $endgroup$
    – robjohn
    Aug 8 '13 at 19:38




    $begingroup$
    @Kathy: we still have to show that $a+1$ doesn't have order $2$. We could argue that if $(a+1)^2=1$ and $(a+1)^3=-1$, then $a+1=-1$; that is, $a=-2$. Since $a^3=1$ we have that $-8equiv1pmod{p}$, which implies that $pmid9$, whence $p=3$. Since there are no elements of order $3$ mod $3$, we could stop. Personally, I think the argument in my answer is simpler.
    $endgroup$
    – robjohn
    Aug 8 '13 at 19:38












    $begingroup$
    Is there a problem with this answer? Just wondering if the downvote means something needs fixing.
    $endgroup$
    – robjohn
    Jan 1 at 19:45




    $begingroup$
    Is there a problem with this answer? Just wondering if the downvote means something needs fixing.
    $endgroup$
    – robjohn
    Jan 1 at 19:45











    1












    $begingroup$

    ord_$pa=3iff p$ divides $(a^3-1)=(a-1)(a^2+a+1)$



    But $pnotmid (a-1)$ as ord_$pa=3 $



    $displaystyleimplies a^2+a+1equiv0pmod p$
    $displaystyleiff a(a+1)equiv-1pmod p$



    Method $1:$



    $implies a^3(a+1)^3equiv-1pmod pimplies (a+1)^3equiv-1$ and $(a+1)^6equiv1$



    $implies $ord_$p(a+1)mid 6$ but does not divide $3$



    If ord_$p(a+1)mid 2, (a+1)^2equiv1pmod p$ and $a^2(a+1)^2equiv1pmod pimplies a^2equiv1$ which is impossible as ord$_pa=3$



    $implies $ord$_p(a+1)=6$



    Method $2:$



    $displaystyle a(a+1)equiv-1pmod piff a+1equiv (-a)^{-1}pmod p$



    Again as $a^3equiv1pmod p, (-a)^3=-a^3equiv-1pmod pimplies (-a)^6equiv1implies $ord$_p(-a)=6$



    Using this, ord$_p{(-a)^{-1}})=$ord$_p(-a)=6$






    share|cite|improve this answer











    $endgroup$


















      1












      $begingroup$

      ord_$pa=3iff p$ divides $(a^3-1)=(a-1)(a^2+a+1)$



      But $pnotmid (a-1)$ as ord_$pa=3 $



      $displaystyleimplies a^2+a+1equiv0pmod p$
      $displaystyleiff a(a+1)equiv-1pmod p$



      Method $1:$



      $implies a^3(a+1)^3equiv-1pmod pimplies (a+1)^3equiv-1$ and $(a+1)^6equiv1$



      $implies $ord_$p(a+1)mid 6$ but does not divide $3$



      If ord_$p(a+1)mid 2, (a+1)^2equiv1pmod p$ and $a^2(a+1)^2equiv1pmod pimplies a^2equiv1$ which is impossible as ord$_pa=3$



      $implies $ord$_p(a+1)=6$



      Method $2:$



      $displaystyle a(a+1)equiv-1pmod piff a+1equiv (-a)^{-1}pmod p$



      Again as $a^3equiv1pmod p, (-a)^3=-a^3equiv-1pmod pimplies (-a)^6equiv1implies $ord$_p(-a)=6$



      Using this, ord$_p{(-a)^{-1}})=$ord$_p(-a)=6$






      share|cite|improve this answer











      $endgroup$
















        1












        1








        1





        $begingroup$

        ord_$pa=3iff p$ divides $(a^3-1)=(a-1)(a^2+a+1)$



        But $pnotmid (a-1)$ as ord_$pa=3 $



        $displaystyleimplies a^2+a+1equiv0pmod p$
        $displaystyleiff a(a+1)equiv-1pmod p$



        Method $1:$



        $implies a^3(a+1)^3equiv-1pmod pimplies (a+1)^3equiv-1$ and $(a+1)^6equiv1$



        $implies $ord_$p(a+1)mid 6$ but does not divide $3$



        If ord_$p(a+1)mid 2, (a+1)^2equiv1pmod p$ and $a^2(a+1)^2equiv1pmod pimplies a^2equiv1$ which is impossible as ord$_pa=3$



        $implies $ord$_p(a+1)=6$



        Method $2:$



        $displaystyle a(a+1)equiv-1pmod piff a+1equiv (-a)^{-1}pmod p$



        Again as $a^3equiv1pmod p, (-a)^3=-a^3equiv-1pmod pimplies (-a)^6equiv1implies $ord$_p(-a)=6$



        Using this, ord$_p{(-a)^{-1}})=$ord$_p(-a)=6$






        share|cite|improve this answer











        $endgroup$



        ord_$pa=3iff p$ divides $(a^3-1)=(a-1)(a^2+a+1)$



        But $pnotmid (a-1)$ as ord_$pa=3 $



        $displaystyleimplies a^2+a+1equiv0pmod p$
        $displaystyleiff a(a+1)equiv-1pmod p$



        Method $1:$



        $implies a^3(a+1)^3equiv-1pmod pimplies (a+1)^3equiv-1$ and $(a+1)^6equiv1$



        $implies $ord_$p(a+1)mid 6$ but does not divide $3$



        If ord_$p(a+1)mid 2, (a+1)^2equiv1pmod p$ and $a^2(a+1)^2equiv1pmod pimplies a^2equiv1$ which is impossible as ord$_pa=3$



        $implies $ord$_p(a+1)=6$



        Method $2:$



        $displaystyle a(a+1)equiv-1pmod piff a+1equiv (-a)^{-1}pmod p$



        Again as $a^3equiv1pmod p, (-a)^3=-a^3equiv-1pmod pimplies (-a)^6equiv1implies $ord$_p(-a)=6$



        Using this, ord$_p{(-a)^{-1}})=$ord$_p(-a)=6$







        share|cite|improve this answer














        share|cite|improve this answer



        share|cite|improve this answer








        edited Apr 13 '17 at 12:19









        Community

        1




        1










        answered Aug 8 '13 at 18:12









        lab bhattacharjeelab bhattacharjee

        224k15156274




        224k15156274






























            draft saved

            draft discarded




















































            Thanks for contributing an answer to Mathematics Stack Exchange!


            • Please be sure to answer the question. Provide details and share your research!

            But avoid



            • Asking for help, clarification, or responding to other answers.

            • Making statements based on opinion; back them up with references or personal experience.


            Use MathJax to format equations. MathJax reference.


            To learn more, see our tips on writing great answers.




            draft saved


            draft discarded














            StackExchange.ready(
            function () {
            StackExchange.openid.initPostLogin('.new-post-login', 'https%3a%2f%2fmath.stackexchange.com%2fquestions%2f463010%2fshow-that-if-a-has-order-3-bmod-p-then-a1-has-order-6-bmod-p%23new-answer', 'question_page');
            }
            );

            Post as a guest















            Required, but never shown





















































            Required, but never shown














            Required, but never shown












            Required, but never shown







            Required, but never shown

































            Required, but never shown














            Required, but never shown












            Required, but never shown







            Required, but never shown







            Popular posts from this blog

            Can a sorcerer learn a 5th-level spell early by creating spell slots using the Font of Magic feature?

            Does disintegrating a polymorphed enemy still kill it after the 2018 errata?

            A Topological Invariant for $pi_3(U(n))$